IFRAME SYNC
November 2020

Question: How to show the relation $$ J:=\int_0^1 \left(\frac \pi 4+\arctan t\right)^2\cdot \log t\cdot\frac 1{1-t^2}\; dt =-\frac 1{64}\pi^4 $$ (using a "minimal industry" of relations, possibly remaining inside the real analysis)?

So i have found a solution to the problem, it is part of my solution for math.stackexchange.com - questions - 3854736, but not a satisfactory solution. "There should be more", explaining why there is a "clean result" for the integral.

Here, i am not strictly interested in a computational approach. I just want to share this with the community in these days of isolation. Any idea to attack this, or a related integral involving "three log factors" is welcome. (Well, the $\arctan$ is a sort of $\log$ in a sense that i don't want to define closer, see below.) Computations may be safely done "modulo integrals involving two or one log factor". But an illuminating, short way to show the above formula for $J$ would be wonderful.


Motivation: The above relation appeared as i tried to solve the integral posted at the above link:

Calculate $\displaystyle\int_0^{2\pi} x^2\; \cos x \cdot\operatorname{Li}_2(\cos x)\; dx$ .

After several simplifications and substitutions, it turns out that the above integral is related to integrals of the shape

  • $\int_0^1\log t\; R(t)\; dt$ , and
  • $\int_0^1\arctan t\cdot \log t\; R(t)\; dt$ , and
  • $\int_0^1\arctan^2 t\cdot \log t\; R(t)\; dt$ ,
  • and "similar" expressions.

Here $R$ is in each case a (rather simple) rational function. (The more log and/or arctangent factors, the higher the computational complexity.)

I could compute more or less algorithmically most of the the needed integrals to solve the linked problem, all of them but the integral $$ K=\int_0^1\arctan^2 t\cdot\log t\cdot\frac2{1-t^2}\; dt\ , $$ which turned out to be very hard to attack with the methods of real analysis. Computing this integral is more or less equivalent to computing $J$, and the question wants $J$ instead, since we have a "clean formula", so that some speculation about a "clever substitution" may be accepted.

My solution (for $K$) works in complex analysis, the first step is to write $$ \int_0^1 =\int_0^i+\int_i^1\ , $$ then parametrize the first integral using a linear path, the second one using a path on the unit circle.


Some comments: I will say some more words, because the situation is rich in coincidences. Since a numerical evidence is the simplest and shortes way to present (instead of showing how to show), i will use this method to at least list the coincidences. Many equalities below are "equivalent" (modulo computation of integrals of lower complexity) to the formula for $J$.

  • First of all, a numerical experiment using pari/gp delivers some connection between $K$ and a "cousin" of $J$:

      ? 2 * intnum( t=0, 1, atan(t)^2 * log(t) / (1-t^2) )
      %88 = -0.357038604620289042902893412499686912781214141574556097366337
      ? real(intnum( t=0, I, (pi/4 - atan(t))^2 * log(t) / (1-t^2) ))
      %89 = -0.357038604620289042902893412499686912781214141574556097366337
      ? intnum( t=0, 1, (pi/4 - atan(t))^2 * log(t) / (1-t^2) )
      %90 = -0.357038604620289042902893412499686912781214141574556097366337
    

In words: $$ \begin{aligned} K &= \int_0^1\arctan^2 t\cdot\log t\;\frac{2}{1-t^2}\; dt \\ &= \Re \int_0^i\left(\frac \pi 4-\arctan t\right)^2 \cdot\log t\;\frac 1{1-t^2}\; dt \\ &= \int_0^1\left(\frac \pi 4-\arctan t\right)^2 \cdot\log t\;\frac 1{1-t^2}\; dt \ . \end{aligned} $$ Note the integration margins. What happens if we take the integral on $[0,i]$ instead of $[0,1]$ in the $K$-integral? Numerically:

    ? 2 * real(intnum( t=0, i, atan(t)^2 * log(t) / (1-t^2) ))
    %98 = 1.52201704740628808181938019826101736327699352613570971392919
    ? pi^4/64
    %99 = 1.52201704740628808181938019826101736327699352613570971392919

In words: $$ \begin{aligned} K^* &:= \Re\int_0^i\arctan^2 t\cdot\log t\;\frac{2}{1-t^2}\; dt \\ &=\frac 1{64}\pi^4 \\ &= -\int_0^1\left(\frac \pi 4+\arctan t\right)^2 \cdot\log t\;\frac 1{1-t^2}\; dt \\ &=-J\ . \end{aligned} $$

(These observations were leading to the formula for $K$ in loc. cit. .)

  • One idea is to use partial integration in $J$ or $K$. Well, we have for $K$: $$ \begin{aligned} K &= \int_0^1\arctan^2 t\cdot\log t\;\left(-\log\frac {1-t}{1+t}\right)'\; dt \\ &= \underbrace{\int_0^1\arctan^2 t\cdot\frac 1t\cdot \log\frac {1-t}{1+t}\; dt}_{=2K\text{ (why?)}} \\ &\qquad\qquad+ \underbrace{\int_0^12\arctan^2 t\cdot\frac 1{t+t^2}\cdot \log t\cdot \log\frac {1-t}{1+t}\; dt}_{=-K\text{ (why?)}} \ . \end{aligned} $$

  • Note that $\arctan$ is related to the logarithm (over $\Bbb C$), we have the relation (around $0$) $$ \arctan t=\frac 1{2i}\log\frac {1+it}{1-it}\ . $$ The substitution $t=\frac{1-u}{1+u}$ and the formula for $\tan(\arctan 1-\arctan u)$ are giving: $$ \begin{aligned} K &= \int_0^1\arctan^2 t\cdot\log t\;\frac{2}{1-t^2}\; dt \\ &=\int_0^1 \left(\frac\pi2-\arctan u\right)^2\cdot\log\frac {1-u}{1+u}\cdot \frac {du}u\ . \\ &=\int_1^\infty \left(\frac\pi2-\arctan u\right)^2\cdot\log\frac {u-1}{1+u}\cdot \frac {du}u\ . \end{aligned} $$ (Write $\log t=\frac 12\log t^2$ to have the same expression under the integral on $(0,1)$ and on $(1,\infty)$.)

  • Note the fact that the factor $\frac 2{1-t^2}$ is not "random". It is the right one to make things feasible. It is the derivative of $\displaystyle -\log\frac{1-t}{1+t}$, and plugging in $t=iu$ into $\displaystyle \log\frac{1-t}{1+t}$ leads to an expression related to $\arctan u$. And conversely, $\arctan(iu)$ is related to such a logarithmic expression in $u$.



from Hot Weekly Questions - Mathematics Stack Exchange
dan_fulea

In ordinary discourse, when we say that $A$ implies $B$, we shall formalize it by writing the following: $$A\rightarrow B$$ But when we say that $A$ does not imply $B$, we cannot formalize it as the following: $$\neg(A\rightarrow B)$$ Because by material implication, we have the following equivalences for the second formula: $$\neg(A\rightarrow B)\Longleftrightarrow\neg(\neg A\vee B)\Longleftrightarrow A\wedge\neg B$$ But the ordinary meaning of the sentence "$A$ does not imply $B$" is that $B$ does not follow from $A$: if $A$ is true, $B$ is either false or undecidable. I wonder how "$A$ does not imply $B$" is formally expressed in the object language (not at the meta-level). Thanks!



from Hot Weekly Questions - Mathematics Stack Exchange
Fred

Given the following recurrence relation :

$f(n) = 5f(n-1) - 2f(n-2)$ where $f(0) = 0, f(1) = 1$

I need to find out if an integer $F_n$ is present in the sequence in $O(1)$ time and space.

Solving the equation, there are two distinct real roots.

$\phi = \frac{5 + \sqrt17}2$

$\psi = \frac{5 - \sqrt17}2$

Therefore, $F_n = \frac{\phi^n - \psi^n}{\sqrt17}$

Similar to Binet's rearranged formula, I want to solve for $n$ in terms of $F_n$.

Since, $\psi = \frac{2}{\phi}$

$\sqrt17F_n = \phi^n - \frac{2^n}{\phi^n}$

$Or,$

$\phi^{2n} - \sqrt17F_n\phi^n-2^n = 0$

Here I'm not able to find out a solution to express $n$ purely in terms of $F_n$ so that I can calculate the perfect square just like in Binet's formula.



from Hot Weekly Questions - Mathematics Stack Exchange
Rohit Roy Chowdhury

I was looking into the proof that there are only five Platonic solids in Basic Concepts of Algebraic Topology by F.H. Croom at page 29, Theorem 2.7. To clarify,

  • We define a Platonic solid as a simple, regular polyhedron.
  • We define a simple polyhedron to be a polyhedron homeomorphic to the $2$-sphere.
  • We define a regular polyhedron to be a polyhedron whose faces are regular polygons all congruent to each other and whose local regions near the vertices are all congruent to each other.

Using homology theory, one can prove that the Euler formula $V-E+F=2$ must hold for Platonic solids. Then by using Euler's formula and invoking a counting argument, we find that there are five possible tuples $(V, E, F)$. This is a beautiful proof, but I am unsatisfied with a question: How do we know there can't be two non-similar Platonic solids that have the same $(V, E, F)$-tuple?

Almost all sources I've looked at seem to assume it is obvious that two Platonic solids with the same $(V, E, F)$-tuple are similar, and it is not obvious to me.

Does anyone have any suggestions for how to prove this? Alternatively, does anyone know of a reference where this is proved rigorously?


Edit 1: It's not completely clear, but it seems like the definition I used for "regular polyhedra" is different than the one commonly used. Note that I am not assuming any global symmetry, so if any global symmetry is to be invoked, it needs to be proven.

Edit 2: I've been made aware of Cauchy's rigidity theorem, which is proven in, e.g., Proofs From the BOOK by Aigner & Zeigler. One can show that any two Platonic solids that have the same $(V, E, F)$-tuple must be combinatorically equivalent. However, in order for the theorem to apply, we need to show that our Platonic solids are convex. I can't seem to think of any rigorous argument for why the Platonic solids have to be convex.

And actually, you don't need to show that the entire polyhedron is convex. If I'm not mistaken, the proof for Cauchy's rigidity theorem only relies on the vertices of the polyhedron being locally convex. So really it suffices to show the vertices are convex.



from Hot Weekly Questions - Mathematics Stack Exchange
Maximal Ideal

I know there is a result that $S=$ collection of all trace $0$ matrices, and that collection forms a vector space. But I want to prove it independently i.e.
For any $A,B,C,D\in M_n(K)$ we have to find $E,F\in M_n(K)$ such that $(AB-BA)+(CD-DC)=EF-FE$.
But I don't know how to solve this. But the thing I can observe that the above equation gives rise to $n^2$ equation (equating each entries of matrices of both the sides) with $2n^2$ variables (Total number of entries of $E,F$ is $2n^2$).Can this problem be simplified if we choose special kind of $E$ say diagonal matrix.
Edit-(This is valid only for $\Bbb{R}$ or $\Bbb{C}$)
$AB-BA=(A+aI)(B+bI)-(B+bI)(A+aI)$ for all $a,b\in\Bbb{R}$. And there is $a,b$ in $\Bbb{R}$ such that $A+aI,B+bI$ are invertible.
Hence, we can assume $A,B$ to be invertible matrices i.e. $S=\{AB-BA|A,B\in GL_n(K)\}$



from Hot Weekly Questions - Mathematics Stack Exchange
Biswarup Saha

In 2004, mathematicians Edward Frenkel, Dennis Gaitsgory, Kari Vilonen, and Mark Goresky were awarded millions of dollars in the form of a grant by DARPA (Defense Advanced Research Projects Agency). The project being funded was to relate the geometric Langlands program to quantum field theory.

According to DARPA’s Wikipedia page, the agency is “a research and development agency of the United States Department of Defense responsible for the development of emerging technologies for use by the military.” In other words, it seems unlikely that an agency dedicated to researching military technology would fund anything with no chance of being applicable to its goal.

Now anyone who has spent any time trying to understand the mathematics related to the geometric Langlands program knows that it’s a highly abstract field. Although the program has roots in number theory, its central conjectures are certain equivalences of DG categories whose definitions take literal books to write out fully. Even the program’s connections to physics (via S-duality a la Witten) seem too deep to have any real impact on “real world” issues.

My question is as stated in the title. Why is the US military interested in funding research related to highly abstract subfields of math and physics?

Two possibilities spring to mind:

1) DARPA has no understanding of the project; it just knows that it’s a hot topic and hopes that some practical application will come of it in the future

2) There are actual concrete applications the military has in mind but they’re classified.

Any thoughts?

submitted by /u/infinitysouvlaki
[link] [comments]

from math https://ift.tt/2IED3Cn https://ift.tt/eA8V8J

What is the biggest circle that is contained in the region bounded by the graph of the polynomial $f(x) = x(1-x)(2x+1)$ and the x-axis interval $[0, 1]$?

enter image description here

(Here's the thing in Desmos)


Here's what I have tried

Let's denote the center of the circle by $(c_x, c_y)$ and the points of tangency by $(x_k, y_k)$, $k=1,2$. From distance to the x-axis, we know that the radius of the circle is then $c_y$. We can express the tangency by the dot product of the vectors $(x_k-c_x, y_k-c_y)$ and $(1, f'(x_k))$ being zero. So we get the group of equations (all for $k=1,2$)

$$\begin{cases} (x_k-c_x)^2 + (y_k-c_y)^2 &=& c_y^2 \\ x_k-c_x + (y_k-c_y)f'(x_k) &=& 0 \\ f(x_k) &=& y_k \end{cases} $$

Then I tried to solve this group by using Gröbner basis and elimination ideals. I wrote this SageMath code:

f(x) = x*(1-x)(2*x+1)
fd = f.derivative()

R.<x1, y1, x2, y2, cx, cy> = PolynomialRing(QQ, order='lex')
polys1 = [(cx-xk)**2+(cy-yk)**2 - cy**2 for (xk, yk) in [(x1,y1), (x2, y2)]]
polys2 = [(xk-cx)+(yk-cy)*fd(xk)  for (xk, yk) in [(x1,y1), (x2, y2)]]
polys3 = [f(xk)-yk for (xk, yk) in [(x1,y1), (x2, y2)]]
I = R.ideal(polys1+polys2+polys3)
gb = I.groebner_basis()
for poly in gb:
    print (poly)
print (I.dimension())

It gave me a long list of polynomials, the last one being

$$c_x^6 - 2c_x^4c_y^2 + \frac{5}{4}c_x^4c_y + \frac{1}{64}c_x^4 + c_x^2c_y^4 + \frac{3}{4}c_x^2c_y^3 + \frac{3}{16}c_x^2c_y^2 + \frac{1}{64}c_x^2c_y$$

and said the dimension is $1$. I now realize the reason we don't get dimension $0$ has probably to do with the fact that the points $(x_1, y_1)$ and $(x_2, y_2)$ could be the same point(?) So for a solution, the point $(c_x, c_y)$ should be a zero of that polynomial? But is even that correct, since the points could be collapsed and the other side go over the curve??

How to solve this even numerically? I tried with Sage by minimizing the square sum of all those polynomials that should be zero. I also tried adding constraints for the variables to be near the solution that can be seen from the picture but that didn't work either.



from Hot Weekly Questions - Mathematics Stack Exchange
minkbag

I need to compute a limit:

$$\lim_{x \to 0+}(2\sin \sqrt x + \sqrt x \sin \frac{1}{x})^x$$

I tried to apply the L'Hôpital rule, but the emerging terms become too complicated and doesn't seem to simplify.

$$ \lim_{x \to 0+}(2\sin \sqrt x + \sqrt x \sin \frac{1}{x})^x \\ = \exp (\lim_{x \to 0+} x \ln (2\sin \sqrt x + \sqrt x \sin \frac{1}{x})) \\ = \exp (\lim_{x \to 0+} \frac {\ln (2\sin \sqrt x + \sqrt x \sin \frac{1}{x})} {\frac 1 x}) \\ = \exp \lim_{x \to 0+} \dfrac {\dfrac {\cos \sqrt x} {x} + \dfrac {\sin \dfrac 1 x} {2 \sqrt x} - \dfrac {\cos \dfrac 1 x} {x^{3/2}}} {- \dfrac {1} {x^2} \left(2\sin \sqrt x + \sqrt x \sin \frac{1}{x} \right)} $$

I've calculated several values of this function, and it seems to have a limit of $1$.



from Hot Weekly Questions - Mathematics Stack Exchange
Andrey Surovtsev

Good morning.

TL; DR: For a long time I convinced myself that computing science (and in particular AI/data science) was the way, until I realised I dislike how everything is monopolised by the industry, and that CS has become solving very 1st world problems (in a bad way). Would it make sense to try taking another degree/doing a PhD in mathematics?

Here is some (long) background about me to explain how did I arrive at such point:

I am a computing science student who has been dealing with identity crises for a long time. I enrolled in computer science because I thought it was an easy way to find a job and make decent salaries (which is partially true) but I came to realise coding is more and more off-putting.

Back in the days of high school, I used to take part to mathematics competitions and studied some advanced stuff like maths for winter camps in preparation to IMO, despite never reaching that level. During my last year of high school I considered whether to continue computing science (Italian high schools have fixed subjects you can take according to the type of your school, so mine had a lot of cs/swe classes) - for which I started to lose my initial fascination after doing cs-ish stuff for 5 years -, mathematics because I like problem solving a lot, and physics. After some considerations I opted for CS as I had clear ideas about what I could do and learn.

At first I was sceptical about joint degrees, thus I did not apply for a joint CS+Maths degree, and I am regretting it now. I guess it's a cultural thing: in my country we always believed that you should study one thing and do it well, so I could not initially accept the Anglosaxon idea of joint degrees. I thought that, despite interesting, one would have ended up having incomplete education about both.

Fast forward to last year: I went down the rabbit hole of data science and for a while I was quite happy of my decision. Until... well, until I realised data science is clearly applied science. And with that a number of factors come in, for example that much of the stack is proprietary (i.e. runs on non-free software), is being monopolised by a small number of companies and that you cannot really do actual data science without both data and computing resources. Data-wise, you need to perform massive (and unethical) data mining on users to stay on top of the concurrence. Computing-wise, the university has some resources, yes, but they can be nowhere as near as the ones that - say - Google, Nvidia, OpenAI etc. have when they train language models like BERT and GPT-2/3 from scratch. Which in practise means your research will not be truly free until you work for a company that wants to make profit out of your work.

I have worked with people coming from one of the aforementioned companies and it sounds like my worst fears were true: extreme pressure to get things done, intellectually dead applied research (e.g. writing a lot of parsers just to get the Google Assistant to 'understand' the intent of your utterance), actually no freedom of self-defining your work until you become a senior researcher etc.

And, last but not least, the actual 'usefulness'. At first I thought "I am going to do CS to solve real-world problems" like treatments for cancer, optimizing aqueduct planning in developing countries etc. I thought of things like combinatorial optimizations, bioinformatics etc.; however, I ended up doing NLP - the 'easiest' field of ML one can get into at the moment - and realised how little it actually "help" end users. Yes, you can make good information retrieval systems but at the end of the day it mainly serves the purpose of making better and better advertisements. Even if you put DS aside, you quickly realise much of software engineering is mainly about making internal tools, dashboards, websites for the nth corporate etc... Why not other classical CS? Because:

  1. AI is clearly "the future". For example, I considered doing formal verification for a while. FV is a tool humans use to proof check their code by - to simply put it - "convert it" into a theorem and run a proof assistant on it, but if an AI can program (yes, current AI models can program, even if at a toy level) and debug itself then what's the point? It would end up being a technique for Good Old-Fashioned AI (GOFAI) - which, to plainly put it, fails to be AI.

  2. Other fields of CS that are ML-free are at the moment mainly in combinatorics (e.g. stringology and graph theory) or numerical optimization or constraint programming. Now, I can't get to like the second one. About the first one... many cool problems in combinatorics are unfortunately very NP, so basically untreatable. Approximated algos are nice but I don't feel like they are 'challenging' enough. You are not cracking a problem but simply finetuning algorithms to better cover edge cases.

Yes, I am aware I can do Data science on medical data (which is something I am seriously trying to do at the moment). But the problems about availability of data, computing resources and, lastly, actual interests of your employer remain.

So, given that much of CS is about solving very 1st world problems... why not just agreeing on solving very artificial puzzles? But this time, nice problems like the ones I find in mathematics? The small issue here is that my knowledge of maths is a bit rusty: yes I know some bits of real analysis, linear algebra, general topology, number theory, category theory, ring theory etc. but I never fully really wondered what I would like to study if I were to do a PhD. Which is why I think it's unlikely I would apply for a PhD in maths even if I were eligible, as I do not have enough preparation to clearly define a research goal.

But then I question myself: would I be up for doing 3/4 more years of studies at a university for Mathematics? I am 21 now and I am starting to feel like I am wasting my youth. Will I do research as usual? How would I sustain myself during this period? What are the policies for second-degrees students (I am European and study in Scotland, but will likely go back to the mainland because of Brexit - thus some financial advice would be great too).

Is there anyone here who has a similar background, in particular wanted to migrate from the "cool" CS to mathematics? What made you do the switch?

submitted by /u/erolm-a
[link] [comments]

from math https://ift.tt/3eVfbXm https://ift.tt/eA8V8J

Does anyone know how to compute analytically the following integral:

$$\int\limits_{0}^{+\infty}\dfrac{x^2\mathrm{d}x}{e^{x}-1}$$

It should be equal to $2\zeta(3)$ according to Maple. I tried the following using the binomial theorem for negative integer exponents:

$$I = \int\limits^{+\infty}_{0} e^{-x}(1-e^{-x})^{-1}x^2\mathrm{d}x = \int\limits^{+\infty}_{0}\left[\sum_{k=0}^{+\infty}(-1)^k(-1)^ke^{-(k+1)x}\right]x^2\mathrm{d}x=\int\limits^{+\infty}_{0}\left[\sum_{k=0}^{+\infty}(-1)^{2k}e^{-(k+1)x}\right]x^2\mathrm{d}x$$

After another change of variables, $y=(k+1)x$:

$$I = \sum_{k=0}^{+\infty}(-1)^{2k} \frac{1}{(k+1)^3}\int\limits_0^{+\infty} y^2e^{-y}\mathrm{d}y$$

The keen eye might recognize $\int\limits_0^{+\infty} y^2e^{-y}\mathrm{d}y$ as the gamma function, $\Gamma(3)=(3-1)!=2$. This, together with a slight nudge to the bottom limit of the summation we can rewrite things as:

$$I = \Gamma(3)\sum_{k=1}^{+\infty} \dfrac{(-1)^{2k}}{k^3}$$

And i see immediately (since the beginning in fact...) an infinite sum that makes me troubles and i can't get rid of. I tried to found if i did any trivial error but i'm focusing since to many hours to found it. That's why I need an external view to point me out my obvious error.

Thanks in advance for your help



from Hot Weekly Questions - Mathematics Stack Exchange
Vincent ISOZ

I got my B.S in Mathematics back in May with the original intention of getting into actuarial sciences, but I fell out of love for that field because I found the self-teaching required for the exams to be too stressful, and I was not disciplined enough to go that route. Unfortunately, I came to this epiphany late into my undergrad, so I didn’t have much of a backup plan, but I still loved math (still do) and I did not have any money remaining in scholarships, so I finished out the semester and moved out to NYC to live with my brother.

I’ve been applying to all sorts of math related jobs since then, with a heavy preference towards analyst and trading roles, but I’ve opened up my horizons to teaching and tutoring roles even though I always insisted I would not teach. Its been 6 months and I’ve only had two serious interviews, but only one if you don’t count a second phone assessment as a “serious” interview. I’ve also had two other interviews for office clerk positions, but those fell though, even though I thought they went great. At this point, I still don’t have a full-time job, only a part-time seasonal job at a department store that ends in December. And I can only sustain myself on my savings for the next three or four months (maybe).

With that said, I am getting really depressed about my financial state. If I run out of money, the only option I can think of is maybe joining the army? I’ve read that they have non-combat roles suited toward people with a mathematics background, but the thought of leaving my online friends worries me, not to mention I would be leaving my brother here alone in the middle of our lease. Working a clerical job as an entry level position would suit me well, but I’m not getting any luck scoring interviews for those either. Are there any other job titles that are good entry level positions for someone like myself? Is there anything I can be doing now to support myself aside from continuing to apply to jobs?

Posting this to r/math because I feel like you guys will probably have more relevant knowledge for someone with a background like my own, but if you think I should throw this over to some other subreddits for advice I can do that.

submitted by /u/shenmoola
[link] [comments]

from math https://ift.tt/2Is1d33 https://ift.tt/eA8V8J

Question: A not uncommon calculus mistake is to believe that the product rule for derivatives says that $(fg)'=f'g'$. If $f(x)=e^{x^2}$, determine, with proof, whether there exists an open interval $(a,b)$ and a nonzero differentiable function $g$ defined on $(a,b)$ such that this wrong product rule is true for $x$ in $(a,b)$.

Is my solution correct? Is there anything I need to improve?

My solution:

We're trying to find a function $g$ such that $(fg)'=f'g'.$ Using the product rule, we get $$f'g + fg' = f'g'.$$ Plugging in both $f(x)=e^{x^2}$ and $f'(x)=2xe^{x^2},$ we get $$2xe^{x^2}g + e^{x^2}g' = 2xe^{x^2}g'.$$ Simplifying and canceling $e^{x^2},$ we get $$\frac{g'}{g} = \frac{2x-1}{2x} = 1 + \frac{1}{2x-1}.$$ Taking the integral of both sides, $$\int \frac{dg}g = \int 1+ \frac{1}{2x-1} \, dx.$$ $$\log|g| = \frac{\log|2x-1|}{2} + x + C.$$ $$|g| = e^{\frac{\log|2x-1|}{2}}e^xe^C.$$ Letting $e^C=K,$ we get $$\boxed{g = Ke^x\sqrt{|2x-1|}},$$ Where K is a constant greater than 0. Therefore, on any interval $(a,b)$ that does not contain the value of $\frac{1}{2},$ there exists nonzero differentiable function $g$ defined on $(a,b)$ such that $(fg)'=f'g'$ is true for $x$ in $(a,b)$.



from Hot Weekly Questions - Mathematics Stack Exchange
CSS Jowoo

Rather than simulating blackjack, anyone can directly compute the expected values and overall strategy of blackjack directly using some applied mathematics. To build such analytical system, finding and solving some certain criteria is necessary, developing each algorithm to correctly solve for the optimal strategy.

What is prompting me to post this is to try to better understand a fundamental concept in blackjack : splitting. If it wasn't for this player option, blackjack would be much simpler to solve, but much boring to play (in my own normative opinion!) Therefore, developing a splitting algorithm is needed alongside solving the nitty-gritty maths associated with it.

I'll try my best to offer some maths to better express what it is I am trying to do; I will also be mixing descriptions of certain algorithms that would be ran on a computer in textual form.

1.) Computing the overall expected value for some strategy

The first thing I want to elucidate is how we can compute the overall strategy for blackjack. We can do this by way of computing E[X] as

E_opt[X|Y, j] = Σ[i = j; k] P[k|Y] * E_opt[Y|Y+k, k]; k < 11

That is, what is the conditional expectation of X given Y as the sum of the sum of each weighted conditional expectation for k cards drawn, all cards less than 11 points? Now, there is a problem. Assuming you evaluate this, we will go on for an infinite amount of time as we are not bounded by some restriction. Therefore, using the rules of blackjack, a hand cannot go above 21 points assuming that : a.) there are no aces that are 11 points that can be reduced to one (the soft hand rule), and b.) if there is an ace that is soft reduce it.

Taking these two rules we can compute the point total of hand Y given:

If a given hand is 11 or less and has an ace (a point of 1), add 10 points to the sum; otherwise, sum the points as is.

Now, if we were to enumerate the above E_opt equation and use the restriction above, how can one solve to find each unique subset plus the number of said subsets? We could simply have a computer do it, but is there a way to solve for these question? Is the above equation an accurate representation/description of an enumeration algorithm for finding/counting player hand and deck subsets?

2.) Finding an algorithm for splitting pair cards

The next step is to compute the expected value for splitting pair cards up to some k hands (usually up to 4, but we can do 3, or 2 as well.) The first thing we need to so is enumerate each subset for each split hand we compute. Say for example you are splitting a pair of 6's {66}. You split the hand to create two unique hands {6x, 6x}. You then enumerate the subset of player hands for each 6 and compute the overall expected value for splitting. You can do this up to 4 times : {6x, 6x, 6x} and {6x, 6x, 6x, 6x}.

The question remains is: how can we use the E_opt[X] equation above to find all subsets for each split, the number of subsets, as well as compute the optimal splitting expectation. Also take into account that sometimes, we may not get a split card as our first card as well as not being able to completely split up to our specified number of cards!

I apologise in advance if this is not clear enough for you. I sometimes have a difficult time expressing what I need in writing. I would also like to apologise for the lack of proper mathematical reasoning. My highest level maths I took was calc in my early college years.

TL;DR : How can I use mathematics to both find and count player/deck subsets for the game of blackjack as well as solving to find a splitting algorithm using maths to, again, find and count subsets as well as computing proper expected values.

submitted by /u/huskyShad0w
[link] [comments]

from math https://ift.tt/3nchKr0 https://ift.tt/eA8V8J

Question - Given a randomly generated set of binary numbers, what is the probability that all of them, when XORed with each other, yield $0$?

Additional information -

  1. All of the binary numbers in the set are different from each other. As an example, $\{0001, 0010, 0011\}$ is a valid set, but not $\{0001, 0010, 0010\}$ or $\{0010, 0010, 0010\}$.

  2. All of the binary numbers have equal length which is some given. For example, one could have a given length of 4, meaning numbers such as $00001$ and $010$ are not allowed.

  3. The length of the set is a given, its members are randomly generated.

  4. Each possible valid binary number has an equal likelihood of being generated.

Context - For a school math project, I decided to analytically investigate the average percentage of errors an error correction code can correct given a data set and error rate. This question I am asking it a part of said project that I am unable to solve. If necessary, I can provide additional context.

I already know how to do this problem if restriction 1 is lifted, but did not really know how to proceed. I visualized the randomly generated data set as being one number stacked on top of another, kind of how we write when adding large numbers on paper by hand, and then taking my answer to be the probability that each column of bits had an even number of $1$s.



from Hot Weekly Questions - Mathematics Stack Exchange
Nitin Singhal

Consider the matrix $A$ given by $A=I-\alpha vv^{T}$ with $v\neq0$ and $v\in\mathbb{R}^{n}$ and $\alpha\neq0$. we want to show that there are two distinct eigenvalues $\lambda_{1},\lambda_{2}$ to be found with their corresponding eigenvectors $x_{\lambda_{1}}$ and $x_{\lambda_{2}}$.

My attempt : By definition, we have that $Ax=\lambda x$ thus : $$ (I-\alpha vv^{T})x=x-\alpha vv^{T}x=x-(\alpha v^{T}x)v $$ One can easily notice that $v$ is nothing but a scalar multiple of $x$ that is to say $x=\beta v$ and thus we have that for $x=v$ we get : $$ Av=(1-\alpha v^{T}v)v $$ Thus, an eigenvalue of $A$ is $\lambda_{1}=1-\alpha v^{T}v$. I am unable to find the second eigenvalue nor the corresponding eigenvectors. I would truly appreciate help as I am lost in the process.



from Hot Weekly Questions - Mathematics Stack Exchange
WiWo

We have had many questions here about the divisibility of $\binom{n}{k}$, many of them dealing with divisibility by powers of primes, or expressions involving the $\textrm{gcd}(n,k)$ (I originally gave several more examples but took TheSimpliFire's advice to shorten the list, and many other examples can still be found by looking at this question's edit history):

The topic has also lead to some interesting research papers recently:

There's also many associated theorems:

However I've come across a related problem which is expressed completely in the title, and is remarkably not covered by the above extensive body of literature. In MathJax, if $s>0$ is an integer (let's also make it the largest one for which $2^s$ divides $n$), under what conditions of $k$ do we have the following:

$$ 2^s \mid n \implies 2^s \left\vert \binom{n}{k} \right. . \tag{1} $$

Since $\binom{n}{k} = \frac{n}{k} \binom{n-1}{k-1}$ and $2^s \mid n$ we are left with determining when $\frac{q}{k} \binom{n-1}{k-1}$ is an integer ($q$ being the result when $n$ is divided by $2^s$).

The implication works for a remarkable number of cases, but not always (for example if $\binom{n}{k}$ is odd).



from Hot Weekly Questions - Mathematics Stack Exchange
user1271772

Suppose that when you submit a job application you have a probability of $0.1$ to receive an interview, and a job interview results in a job offer with probability $0.2$. Also assume you only submit one application at a time (i.e. you wait to know if you have been rejected from the job, either at the application stage or the interview stage before applying to another job).

What is the probability of getting your first job offer after submitting at most 3 job applications?


Attempt: Either you get a job offer after your first, second, or third application.

  • The probability of getting an offer after your 1st application is $(0.1)(0.2)=0.02$.
  • The probability of getting an offer after your 2nd application is $(1-0.02)(0.1)(0.2)=0.0196$
  • The probability of getting an offer after your 3rd application is $(1-0.02)(1-0.02)(0.1)(0.2)=0.0192$

Thus, $0.02+0.0196+0.0192=0.0588$

I don't think my approach is correct, it doesn't make sense to me that you have a higher probability of getting an offer on your first application than you do on your 2nd or 3rd. I'm stumped as to what the mistake is that I am making.



from Hot Weekly Questions - Mathematics Stack Exchange
diteni1927

Hello, I'm a Computer Science and Mathematics major who concentrate in cyber security. I'm pretty good at programming, currently focusing on improve my math. I'm not sure what are important math classes to take for modern day, because technology keep changing. Are Cryptography more focus on applied math, computational science math, or pure math?

submitted by /u/Ashamed-Sandwich8518
[link] [comments]

from math https://ift.tt/35n5UEk https://ift.tt/eA8V8J

Is burnout common? I'm currently in my 5th semester of undergrad, and I'm taking 4 math classes. At the beginning of the semester I was following along pretty diligently, but at this point I've seemingly completely lost interest in anything mathematics related. I'm questioning whether or not I should even still apply to graduate school, since everyone I've talked to says that this is the life of a graduate student.

Has anyone else felt this way? I know of some kids who still are eager to wake up and do 10 hours of math, while at this point I just learn enough to get through my classes. If, however, this is a sign that a career in math might not be for me, then that wouldn't be a bad thing, at least. It's good to find that out.

submitted by /u/bounded_variation
[link] [comments]

from math https://ift.tt/36u2UW8 https://ift.tt/eA8V8J

I have six different PCAs (principal components analysis) on six different datasets that all measure the same eight variables. I'm trying to see how similar the PCAs are between each of these datasets. So far I've computed pairwise angles between each of the PC1s, but is there a way to compare datasets with one another using the full matrices (i.e. all 8 dimensions)? Apologies, my background is not at all in mathematics. I hope this makes sense.

submitted by /u/Sarahsea6
[link] [comments]

from math https://ift.tt/3ngpYOH https://ift.tt/eA8V8J

I have the following elementary problem/question that I do not know how to tackle. It comes with a "math-olympiad-flavor" but I suspect it may be much more difficult than an high-school olympiads problem.

Let $\mathscr{P}$ be a simple polygon (not necessarily convex) in the plane, and such that the number of sides of $\mathscr{P}$ is an odd number $2k+1$.

For each vertex $v$ of $P$ there is an intuitive notion of opposite side (because the number of sides is odd), just label the sides counterclockwise at $v$ from $1$ to $2k+1$, and it would be the one with the label $k+1$.

Now suppose that for each vertex and its corresponding opposite side I consider the triangle the determine in the plane. Let us call $\Delta_v$ this triangle and call it a central triangle.

Is the following:

$$ \mathscr{P} \subseteq \bigcup_{v\in \mathscr{P}} \Delta_v$$

always true? In other words, if one paints all central triangles is it true that the whole polygon is then painted?



from Hot Weekly Questions - Mathematics Stack Exchange
Luis Ferroni

We will denote by $\mathbb{K}$ one of the fields $\mathbb{Q}, \mathbb{R}$ or $\mathbb{C}$. On $\mathbb{K}\times \mathbb{K}$ we define the following equivalence relation: $$(a,b)\equiv (a', b') \iff \exists (q,\alpha)\in \mathbb{K}^{*}\times \mathbb{K} \text{ such that } \begin{cases} a=q^2a'+\alpha^2-b\alpha \\ b=qb'+2\alpha \end{cases}.$$ We wish to determine the quotient set $\mathbb{K} \times \mathbb{K}/\equiv$ $\space$ for all $\mathbb{K}$s.
This problem was an extra problem in my abstract algebra class (don't worry, this isn't an attempt to cheat, I am posting this a week after the solution was due to be sent) and I kind of got stuck when it comes to $\mathbb{K}=\mathbb{Q}$.
For $\mathbb{K}=\mathbb{C}$, the things were nice and easy, because the original question asked me to prove that $\mathbb{C}\times \mathbb{C}/\equiv$ is equal to $\{\hat{(0,0)}, \hat{(0,1)}\}$ and this can be checked through (tedious) direct computations.
For $\mathbb{K}=\mathbb{R}$, a friend came up with the idea of expressing $\alpha$ from the second equation and then substituing it in the first one. This gives us the following equivalent characterisation of the equivalence relation: $$(a,b)\equiv (a', b') \iff \exists q\in \mathbb{K}^{*} \text{ such that } 4a+b^2=q^2(4a'+b'^2) \space (*).$$ (notice that this works for all $\mathbb{K}$s, the case $\mathbb{K}=\mathbb{C}$ can be solved much easier by using this, but I didn't really need to think that much for that one since direct computations worked in my context)
For real numbers, this rewrites as $(a,b)\equiv (a', b') \iff \operatorname{sgn}(4a+b^2)=\operatorname{sgn}(4a'+b'^2)$.
As a result, there will be three equivalence classes: the parabola $4x+y^2=0$, its interior and its exterior. A representative for each of these are, respectively, $(0,0), (-1,0)$ and $(0,1)$, so $\mathbb{R}\times \mathbb{R}/\equiv \space = \{\hat{(0,0)}, \hat{(-1,0)}, \hat{(0,1)}\}$.
For $\mathbb{K}=\mathbb{Q}$, the things get pretty nasty by this approach. In this case, $(*)$ rewrites as $(a,b)\equiv (a',b') \iff \sqrt{\frac{4a+b^2}{4a'+b'^2}}\in \mathbb{Q}$ and I haven't been able to make any further progress.



from Hot Weekly Questions - Mathematics Stack Exchange
Alexdanut

QUESTION: Let $f:A \rightarrow \mathbb{R}$ be a limited function and let $P$ be a partition of the block $A$ ($A$ is a block in $\mathbb{R}^m$). Then $f$ is integrable $\iff$ for every sub-block $B$ we have that the function $f|_{B}$ is integrable and in this case, $$\int_{A}f=\sum_{B}\int_{B}f|_{B}$$.

REMARK: The professor allowed us to use the following concepts:

  1. Proposition: Let $P_0$ be an arbitrary partition of the block $A$. In order to consider the upper and lower integrals of the limited function $f:A \rightarrow \mathbb{R}$, we just need to consider partition refinements of $P_0$. That is, we have $$\underline\int_{A} f(x) dx= \underset{P\supset P_0}{sup} s(f; P)$$ and $$\overline\int_{A} f(x) dx= \underset{P\supset P_0}{inf} S(f; P)$$
  2. Theorem: The limited function $f: A \rightarrow \mathbb{R}$ is integrable $\iff$ for every $\epsilon>0$ it is possible to find a partition $P$ of the block $A$ such that $$\displaystyle\sum_{B\in P} \omega_{B}\cdot vol B<\epsilon$$ Where $\omega_{B}$ is the set of the oscillations, i. e., $$\omega_{B}:= sup\{|f(x)-f(y)|; x, y \in B\}$$

MY ATTEMPTY:

$(\Longrightarrow)$ Let $f: A \rightarrow \mathbb{R}$ be a limited function and let $P$ be a partition of the block $A$. Suppose that $f$ is integrable then $\forall \epsilon >0$ it is possible to obtain an partition $P=P_1 \times \cdots \times P_n$ of $A$ such that $\displaystyle\sum_{B\in P} \omega_B \cdot \text{vol}B <\epsilon$, where $B$ are blocks in $P$. Once $B$ are sub-blocks of $A$, let $P_0$ be an partition of $B$. Therefore for every limited function $f|_{B}$ we just need to consider the refinement partitions of $P_0$. Indeed, let $B=\displaystyle\Pi_{i=1}^{n}[b_i, c_i] \subset A$ then for every $i=1, \cdots, n$ lets define $Q_i= P_i\cap[b_i, c_i]$ from this we have a new partition $Q = Q_1 \times \cdots \times Q_n$ of $A$ that is a refinement of $P$ and, furthermore, the blocks of $Q$ are contained in $B$ makes a partition $P_0$ of $B$. Thus $$\underbrace{\displaystyle\sum_{B'\in P_0}\omega_{B'}\cdot \text{vol} B'}_{(I)}\leq\displaystyle\underbrace{\sum_{B\in P}\omega_{B}\cdot \text{vol} B<\epsilon}_{(II)}$$ $(I) \subset (II)$ therefore $f|_{B}$ is intagrable.

$(\Longleftarrow)$ We just need to consider $P=P_1 \times \cdots \times P_n$ as a partition of the block $A$ and we also need to consider that this partition is a composition of the block $A$ in sub-blocks like $B=I_1 \times \cdots \times I_n$ where every $I_j$ is an interval of the partition $P_j$, where every sub-block $B$ is the block of partition $P$, i.e., $B\in P$. So, writting $A=\displaystyle\bigcup_{i=1}^{n}B_i$ and remembering that every $f|_{B}$ is integrable. Note that if $P_i$ is a partition of $B_i$ we can consider $Q=\displaystyle\sum_{i=1}^{n}P_i$ as an refinement partition of $P$ thus $f:A \rightarrow\mathbb{R}$ is integrable.

Now we just need to show that: $$\int_{A} f \leq \displaystyle\sum_{B \in P} \int_{B} f|_{B}$$.

In $f:A \rightarrow \mathbb{R}$ considering the partition $P$ of the block $A$ we just need to consider refinement partitions of $P$, let $Q$ be an arbitrary partition of the block $A$ we can consider, for instance $P_0= P+Q$. It follows from upper integration definition that $$s(f, P)=\displaystyle\sum_{B \in P} m_B(f)\cdot \textbf{vol}B= \displaystyle\sum_{B \in P} m_{B}(f|_{B}) \cdot \textbf{vol} B$$. Then, for every $B$ we consider $B' \subset B$, the sub-blocks of $B$ resultants of the refinement of $P$, and $B=\bigcup B'$. Therefore, \begin{align*} \int_{A} f = \displaystyle sup_{P_0\supset P} s(f, P_0)& = sup \left(\displaystyle\sum_{B\in P}m_{B}(f|_{B}) \cdot \textbf{vol} B\right)\\ & = sup \left(\displaystyle\sum_{B\in P}m_{B}(f|_{B}) \displaystyle\sum_{B'\subset B} \textbf{vol} B'\right)\\ & = sup \left(\displaystyle\sum_{B\in P}\displaystyle\sum_{B'\subset B}m_{B}(f|_{B}) \textbf{vol} B'\right)\\ & \leq sup \left(\displaystyle\sum_{B\in P}\displaystyle\sum_{B'\subset B}m_{B'}(f|_{B}) \textbf{vol} B'\right)\\ & = \displaystyle\sum_{B\in P} sup \left(\displaystyle\sum_{B'\subset B}m_{B'}(f|_{B}) \textbf{vol} B'\right)\\ & = \displaystyle\sum_{B\in P} \underline{\int_{B}} f|_{B}\\ & = \displaystyle\sum_{B\in P} \int_{B} f|_{B} \end{align*} Thus, $$\int_{A} f \leq \displaystyle\sum_{B\in P} \int_{B} f|_{B}$$

Similarly, we can show for upper sum, and obtain $$\int_{A} f \geq \displaystyle\sum_{B\in P} \int_{B} f|_{B}$$ And finally, conclude $$\int_{A} f = \displaystyle\sum_{B\in P} \int_{B} f|_{B}$$

MY DOUBT: Would you help me to improve my answer? Specialy in this $(\Longleftarrow)$ way.



from Hot Weekly Questions - Mathematics Stack Exchange
Silvinha

AWM Social

Friday, November 13th

5 PM EST

 

AWM (Association for Women in Mathematics) is hosting our own virtual social on Friday, November 13th at 5 PM EST. We will chat about the Spring 2021 Course Catalog, and we encourage all students interested in taking any course in the department to come. This is a great and casual opportunity to meet and hear from current students about the Mathematics and Statistics courses that will be offered in the spring and to get answers for any questions you may have!

 

Please join us through this Zoom link: https://williams.zoom.us/j/99579131155



from Blog – Mathematics & Statistics
Mihai Stoiciu

Im volunteering at a program at my university where we can teach free 1 day classes to students about whatever topic we like. I chose to do a class that tries to get kids interested in math for math's sake. The students are in high school so I'm hoping they are old enough to understand some of the underlying beauty of mathematics.

Some of the examples I want to include are the role of the golden ratio in the formation of sunflowers and the fact that a rope held up at two ends forms a hyperbolic cosine.

Does anyone have any ideas that get at the heart of the beauty of math while being understandable to an audience that has likely only taken algebra 1 and geometry?

submitted by /u/riskyrainbow
[link] [comments]

from math https://ift.tt/3lrK5ck https://ift.tt/eA8V8J

      During these days of classifying people and points of view, my thoughts turn again and again to Venn Diagrams and I am then reminded of a thoughtful and imaginative poem (by Pennsylvania poet and professor Marjorie Maddox) that I first read long ago -- and I offer it here:  

Learn about Venn Diagrams here

Venn Diagrams     

          by Marjorie Maddox   

There, stuck in that class,
chalking circles on a board 
       so high your toes ached,
an inch of sock exposed,
all for the sake of subsets,
        intersection.
That teacher with the tie too bright for day,
wide as your fingers spread  

Read more »

from Intersections -- Poetry with Mathematics
https://ift.tt/2UiMJW4
noreply@blogger.com (JoAnne Growney)

Aside from $1!\cdot n!=n!$ and $(n!-1)!\cdot n! = (n!)!$, the only nontrivial product of factorials known is $6!\cdot 7!=10!$.

One might naturally associate these numbers with the permutations on $6, 7,$ and $10$ objects, respectively, and hope that this result has some kind of connection to a sporadic relation between such permutations - numerical "coincidences" often have deep math behind them, like how $1^2+2^2+\ldots+24^2=70^2$ can be viewed as an ingredient that makes the Leech lattice work.

The most natural thing to hope for would be a product structure on the groups $S_6$ and $S_7$ mapping to $S_{10}$, but as this MathOverflow thread shows, one cannot find disjoint copies of $S_6$ and $S_7$ living in $S_{10}$, so a product structure seems unlikely.

However, I'm holding out hope that some weaker kind of bijection can be found in a "natural" way. Obviously one can exhibit a bijection. For instance, identify the relative ordering of $1,2,\ldots 7$ in a permutation of size $10$, and then biject $_{10}P_{3}=720$ with $S_6$ in some way. But I'd like to know if there is a way to define such a bijection which arises naturally from the permutation structures on these sets, and makes it clear why the construction does not extend to other orders.

I tried doing something with orderings on polar axes of the dodecahedron ($10!$) and orderings on polar axes of the icosahedron ($6!$), in the hopes that the sporadic structure and symmetry of these Platonic solids would allow for interesting constructions that don't generalize, but ran into issues with the dodecahedron (sequences of dodecahedral axes aren't particularly nice objects) and the question of how to extract a permutation of length $7$.

I'm curious if someone can either devise a natural bijection between these sets or link to previous work on this question.



from Hot Weekly Questions - Mathematics Stack Exchange
RavenclawPrefect

Context
There are two different Wiener processes $W_t$ and $V_t$. It's known that they are independent. Additionally, we are given with third Wiener process $B_t$ that is given by the formula $$B_t = aW_t+bV_t, \quad \quad a^2 + b^2= 1.$$

Problem
Find the limit in $L^2$ of $$S_n = \sum_{i=1}^n\left[B_{it/n} - B_{(i-1)t/n}\right]\left[V_{it/n} - V_{(i-1)t/n}\right]$$
as $n$ tends to infinity.

My ideas
I assume that this is the type of task where we need to calculate the expected value and the variance. As the latter tends to $0$ (it should), we can say that the desired limit is the expected value. The issue is that it's very overextended work to calculate the $E(S_n)$.

Let $W_{it/n} = X_i$ and $V_{it/n} = Y_i$. We have
$$\Bbb E(S_n) = \Bbb E\sum_{i=1}^n [aX_i + bY_i - aX_{i-1} - bY_{i-1}][Y_{i} - Y_{i-1}]$$ which can be written as $$\sum \Bbb E\bigg( aX_iY_i + bY_i^2 - aX_{i-1}Y_i - bY_{i-1}Y_i - aX_iY_{i-1} - bY_iY_{i-1} + aX_{i-1}Y_{i-1} + bY_{i-1}^2\bigg).$$ Next calculations confuse me (what is $\Bbb E(X_i Y_{i-1})$?) Is it zero? And the main question how to calculate the variance?

If I'm not mistaken, $\Bbb E(S_n) = nb \to \infty$, so we don't need variance. Am I right?



from Hot Weekly Questions - Mathematics Stack Exchange
student

Charlotte Scott Centre for Algebra

On Tuesday the 10th of November 2020, Anitha Thillaisundaram spoke at the first series of online meetings, called “New directions in group theory and triangulated categories” and hosted virtually by the University of Manchester. Anitha spoke on Amit’s conjecture for words in finite nilpotent groups, which is joint work with Rachel Camina and Ainhoa Iniguez. The abstract of her talk is as follows:

Abstract:Let w be a word in k variables. For a finite nilpotent group G, a conjecture of Amit states that N_w(1) geq |G|^{k-1}, where N_w(1) is the number of k-tuples (g_1,…,g_k) of elements in G such that w(g_1,…,g_k)=1.This conjecture is known to be true for finite groups of nilpotency class 2. In this talk, we consider a generalized version of Amit’s conjecture and discuss known results.

View original post



from Maths & Physics News
Evgeny Khukhro

Act One:  Take a look at the pic. What questions come to mind?

Just before Thanksgiving there are competitions all over the world to celebrate cool design, tricky engineering, and to donate a whole lot of food. At the end of the exhibit, all the creations are dismantled and the construction materials (thousands of cans of food) are given directly to soup kitchens, food banks, shelters and senior homes. Canstruction, Inc. now has events in over 200 cities worldwide.

Above is a picture of an exhibit called "Downside Up" which was the winning exhibit from Canstruction, 2010, in New York City.

Act Two:  How many cans does it take to build this structure?  What information do you need to determine this?  How did you determine your solution? What else did you notice that is mathematical?

The Activity: Canstruction.pdf

CCSS: 4.MD.3, 6.EE.1, 5.MD.5, 7.G.6, HSF.LE.2, MP2, MP3, MP7

Act Three: 

  • We have a detailed solution and teacher tip page: Canstruction-solution.pdf and the Word docx if you would like to change the activity: Canstruction.docx
  • You can also find a different solution at the website that covered the event:
    http://www.glenwoodnyc.com/manhattan-living/canstruction-nyc-2010-at-the/.  How they got their solution isn't explained and our count was different but there are some other lovely creations to view.    It might be an interesting discussion as to why in the article above they came to a different solution.
  • Or let students continue their investigation with: Have a Heart.
  • Or let students choose any of the creations in our movie to analyze.



from Yummy Math
Leslie

I'm currently an undergrad and I want to get into an academic job for pure mathematics because I want to do both research and teaching in the field. Particularly, I'm interested in a field related to Algebra. Yet I have undergraduate friends who seem to be super dedicated and passionate in pure math as well. For example, I have a friend, and she clocks in long hours of doing math to the point of being similar to a full-time job. She seems to have received plenty of exposure to math even before she pursued her studies in uni.

My background in mathematics may be different from many other math majors, as I've been exposed to writing formal, rigorous proofs and doing thought-provoking math problems (similar to those seen from competitions), so I have experience in problem-solving, before pursuing uni. Then again, I have this feeling that all the other pure math majors are better than I am, which is what makes me feel like a moron who is currently questioning my self-worth.

I appreciate being with like-minded people, who all love mathematics just like I do, but nevertheless, I can't shake off this internal fear that they will go to top-ranking grad schools and subsequently take my job while I face the possibility of failure to break into academe. I guess this is me being ultracompetitive but it seems that the academic job market is a nightmare (and I can't see myself doing applied math in my life). Nonetheless, I find that this mindset will most likely damage relationships that I have with my pure math friends, as well as my mental and emotional well-being.

submitted by /u/restowl123
[link] [comments]

from math https://ift.tt/35kHiMn https://ift.tt/eA8V8J

Given $a_{n}\geq0$, let $b_{n}$ be rearrangement of $a_{n}$. Show that $\sum b_{n}=\sum a_{n}$

My Proof : Let $A_{n}=a_{0}+a_{1}+\cdots+a_{n}$ and let $B_{n}=b_{0}+b_{1}+\cdots+b_{n}$ be a rearrangements of $a_{n}$. Now we note that the sequence of the partial sums $A_{n}$ and $B_{n}$ are either finite, or either $A_{n}$ or $B_{n}$ approach infinity. Thus, it may either be the case that $A_{n}\leq\displaystyle\lim_{n\to+\infty}B_{n}$ or $B_{n}\leq\displaystyle\lim_{n\to+\infty}A_{n}$. We have that $\displaystyle\lim_{n\to+\infty}B_{n}$ involves all the the sum over all $b_{n}$ and so it involves the sum over $a_{0},a_{1},...,a_{n}$ and same for $\displaystyle\lim_{n\to+\infty}A_{n}$ . Taking the limit $n\to+\infty$ we find that $\displaystyle\lim_{n\to+\infty}B_{n}\leq \displaystyle\lim_{n\to+\infty}A_{n}$ and $\displaystyle\lim_{n\to+\infty}A_{n}\leq\displaystyle\lim_{n\to+\infty}B_{n}$ and thus we have that $\displaystyle\lim_{n\to+\infty}A_{n}=\displaystyle\lim_{n\to+\infty}B_{n}$

Is this a complete proof or is my proof missing some details?



from Hot Weekly Questions - Mathematics Stack Exchange
WiWo

Let $\mathcal{E}$ be a topos with subobject classifier $1\overset{t}{\rightarrow}\Omega$, then I want to show that $\Omega$ is injective.

So let $f:A\rightarrow\Omega$ and $g:A\rightarrow B$ be two maps in $\mathcal{E}$ with $g$ monic. Then I want to show that there exists a map $h:B\rightarrow\Omega$ such that $h\circ g = f$. Notice that by the property of a subobject classifier we have that there exists a unique map $\phi:B\rightarrow\Omega$ such that there is a pullback square $\require{AMScd}$ \begin{CD} A @>{\psi}>> 1\\ @VV{g}V @VV{t}V\\ B @>{\phi}>> \Omega \end{CD} Now I claim that $\phi$ does the job for $h$. But I don't see why $\phi\circ g= f$ (so maybe my claim is wrong). Any help would be appreciated!



from Hot Weekly Questions - Mathematics Stack Exchange
Peter

In this series of posts, we’ll be featuring mathematical podcasts from all over the internet, by speaking to the creators of the podcast and asking them about what they do.

We spoke to Rob Eastaway and Andrew Jeffrey about their new podcast, Puzzling Maths.

Podcast title: Puzzling Maths
Website: puzzlingmaths1.podbean.com
Links: RSS feed
Average episode length: 23 minutes
Recommended episode: Episode 4

Puzzling Maths

What is your podcast about, and when did it start? 

This podcast is about puzzles and the maths of everyday life, away from the context of school. It was partly inspired by the success of Andrew Jeffrey’s daily ‘Learner Drivers’ puzzle slot on BBC 5Live during Lockdown. The first episode went out in August 2020.

Tell us about yourselves – who are you?

We are Rob Eastaway and Andrew Jeffrey – two maths popularisers who want to spread the joy of maths beyond those who are already addicted to the subject.

Who is the intended audience for the podcast? 

This is aimed at that broad cross-section of the general public who are curious about maths and puzzles but would not necessarily regard themselves as mathematicians. The content will also appeal to teachers who are interested in hearing how those outside the classroom perceive and use the subject.

What is a typical episode like?

Our typical episode includes: a couple of quickie puzzles that we set each other; a discussion of unusual ways in which maths has cropped up in our everyday life since the previous podcast; an interview with a special guest to ask how maths features in their profession; and then a more involved puzzle that we leave with the listener to be answered in the next episode.

Why should people listen? Why is it different to other mathematical podcasts?

The podcast is very conversational, and an easy and inclusive way for ‘non-mathematicians’ to discover new mathematical ideas and discover how mathematicians think.

What are some highlights of the podcast so far?

Interviewing BBC radio broadcaster Anna Foster in Episode 3 and asking her why the media often appears to boast about not being good at maths. It’s also fascinating to discover what maths our guests would teach in schools if they were given control of the curriculum for a day.

What exciting plans do you have for the future? 

We want to include as diverse a range of guests as we can: historians, musicians, doctors – everyone has a connection with maths.



from The Aperiodical https://ift.tt/2IgUb1y
Katie Steckles

Charlotte Scott Centre for Algebra

On Wednesday the 11th of November 2020, Matteo Vannacci (University of the Basque Country) will be giving an online seminar at the Charlotte Scott Centre for Algebra at the University of Lincoln. His talk will be at 3.30pm and the details of his talk are as follows:

Title: In search of new hereditarily just infinite groups

Abstract: An infinite group is said to be just infinite if it has noinfinite proper quotient. A just infinite group is hereditarily justinfinite if each of its finite index subgroups is just infinite.
Hereditarily just infinite groups are – as of today – quitemysterious. In this talk we will survey what is know about them and Iwill present some work in progress with G. Fernandez-Alcober and M.Noce about hereditarily just infinite groups representable asself-similar subgroups of rooted trees.

View original post



from Maths & Physics News
Evgeny Khukhro

A Ramanujan summation is a

technique invented by the mathematician Srinivasa Ramanujan for assigning a value to divergent infinite series

In my case, I'm interested in assigning a value to the divergent series

$$\sum_{n=1}^\infty f(n) \ \ \ \ \ \ \ \text{where}\ \ \ \ f(n)=\sqrt[n]{2}$$

According to the Wikipedia page (and my understanding), the Ramanujan summation is

$$\sum_{n=1}^\mathfrak{R} f(n)=\lim_{N\to\infty}\Bigg[\sum_{n=1}^N f(n)-\int_{1}^N f(t)dt\Bigg]$$

Thus

$$\sum_{n=1}^\mathfrak{R} \sqrt[n]{2}=\lim_{N\to\infty}\Bigg[\sum_{n=1}^N \sqrt[n]{2}-\int_{1}^N \sqrt[t]{2}dt\Bigg]$$

Taking the antiderivative

$$\sum_{n=1}^\mathfrak{R} \sqrt[n]{2}=\lim_{N\to\infty}\Bigg[\sum_{n=1}^N \sqrt[n]{2}-\Bigg(\ln2\Big(\text{li}\ 2-\text{Ei}\frac{\ln2}{N}\Big)+N\sqrt[N]{2}-2\Bigg)\Bigg]$$

Moving some constants outside the limit

$$\sum_{n=1}^\mathfrak{R} \sqrt[n]{2}=2-\ln2\cdot\text{li}\ 2+\lim_{N\to\infty}\Bigg[\sum_{n=1}^N \sqrt[n]{2}-\Bigg(N\sqrt[N]{2}-\ln2\cdot\text{Ei}\frac{\ln2}{N}\Bigg)\Bigg]$$

It's at this point I'm unsure of how to proceed. I'm not terribly confident what the limit converges to. From my computational estimates up to $N=10^8$, I find that

$$\sum_{n=1}^\mathfrak{R} \sqrt[n]{2}\approx1.6$$

But due to floating point errors or slow convergence, it deviates substantially enough for me to not be confident about any more digits.

I'd like to know if this converges at all, and if it does, is there a (reasonably) closed form / relation to other constants?



from Hot Weekly Questions - Mathematics Stack Exchange
Graviton

Porous numbers are numbers $k$ which are not multiples of 10 such that every m with sum of digits = $k$ and $k$ a divisor of both $m$ and rev($m$) has a zero in its digits. rev($m$) is the digit reversal of $m$ (e.g. rev(123) = 321).

Below 1000 there are only 11, 37, 74, 101 and 121 which fulfill these requirements (see OEIS sequence A337832).

Since $11$ and $11^2$ are porous, I wonder if $11^3$ and eventually all $11^n$ might be porous as well.



from Hot Weekly Questions - Mathematics Stack Exchange
Rüdi Jehn

Recently I came up with a problem regarding Fibonacci numbers:

For which $N$ is it possible to arrange all whole numbers from $1$ to $N$ in such a way that every adjacent pair sums up to a Fibonacci number?

I have manually tested a bunch of cases and I was also able to prove almost every case. The results that I was able to prove are the following:

  • If $N$ is a Fibonacci number or exactly one less than a Fibonacci number, I was able to prove that an arrangement exists. I used an argument by induction to achieve this.
  • If $F_k+2 \leq N \leq F_{k+1} -3$ , it is completely impossible, because the numbers $F_k$, $F_k + 1$ and $F_k + 2$ have only one possible pair and therefore have to be at the end of the arragement. This obviously gives a contradiction, because arrangements only have two ends (the first number and the last number).

The cases I was not able to solve are $N=F_k+1$ and $N=F_k-2$. My theory is that $N=9$ is the only working case of the form $N=F_k+1$, and $N=11$ the only working case of the form $F_k-2$. I expect every other $N$ of these two forms to be impossible.

Does anybody know a full proof to this problem or maybe the name of the official theorem (if this exists)?



from Hot Weekly Questions - Mathematics Stack Exchange
Rubenscube

Contact Form

Name

Email *

Message *

copyrighted to mathematicianadda.com. Powered by Blogger.
Javascript DisablePlease Enable Javascript To See All Widget

Blog Archive